Menu Close

If-n-1-k-2-k-3-1-k-3-1-n-Solve-for-complex-numbees-z-4-3z-3-z-2-3z-1-0-




Question Number 188224 by Shrinava last updated on 26/Feb/23
If   Ω = Σ_(n=1) ^∞  (Π_(k=2) ^∞  ((k^3  − 1)/(k^3  + 1)))^n   Solve for complex numbees:  z^4  + 3z^3  + Ωz^2  + 3z + 1 = 0
$$\mathrm{If}\:\:\:\Omega\:=\:\underset{\boldsymbol{\mathrm{n}}=\mathrm{1}} {\overset{\infty} {\sum}}\:\left(\underset{\boldsymbol{\mathrm{k}}=\mathrm{2}} {\overset{\infty} {\prod}}\:\frac{\mathrm{k}^{\mathrm{3}} \:−\:\mathrm{1}}{\mathrm{k}^{\mathrm{3}} \:+\:\mathrm{1}}\right)^{\boldsymbol{\mathrm{n}}} \\ $$$$\mathrm{Solve}\:\mathrm{for}\:\mathrm{complex}\:\mathrm{numbees}: \\ $$$$\mathrm{z}^{\mathrm{4}} \:+\:\mathrm{3z}^{\mathrm{3}} \:+\:\Omega\mathrm{z}^{\mathrm{2}} \:+\:\mathrm{3z}\:+\:\mathrm{1}\:=\:\mathrm{0} \\ $$
Answered by aleks041103 last updated on 27/Feb/23
((k^3 −1)/(k^3 +1))=(((k−1)(k^2 +k+1))/((k+1)(k^2 −k+1)))  Π_(k=2) ^m ((k^3 −1)/(k^3 +1))=Π_(k=2) ^m ((k−1)/(k+1))Π_(k=2) ^m ((k^2 +k+1)/(k^2 −k+1))  Π_(k=2) ^m ((k^2 +k+1)/(k^2 −k+1)) = ((Π_(k=2) ^m k^2 +k+1)/(Π_(k=2) ^m k^2 −k+1)) =  =((Π_(k=2) ^m k^2 +k+1)/(Π_(k=2) ^m (k−1)^2 +(k−1)+1))=  =((Π_(k=2) ^m k^2 +k+1)/(Π_(k=1) ^(m−1) k^2 +k+1))=((m^2 +m+1)/3)  Π_(k=2) ^m ((k−1)/(k+1))=Π_(k=2) ^m ((k−1)/k)Π_(k=2) ^m (k/(k+1))=  =(1/m) (2/(m+1))=(2/(m(m+1)))  Π_(k=2) ^m ((k^3 −1)/(k^3 +1))=((2(m^2 +m+1))/(3m(m+1)))  ⇒Π_(k=2) ^∞ ((k^3 −1)/(k^3 +1))=lim_(m→∞) ((2(m^2 +m+1))/(3m(m+1)))=(2/3)  ⇒Ω=Σ_(n=1) ^∞ ((2/3))^n =(2/3)Σ_(n=0) ^∞ ((2/3))^n =  =(2/3) (1/(1−(2/3)))=2  z^4 +3z^3 +Ωz^2 +3z+1=0  z^4 +3z^3 +2z^2 +3z+1=0  z=0⇒not sol.  z^2 +3z+2+3(1/z)+(1/z^2 )=0  w=z+(1/z)⇒w^2 =2+z^2 +(1/z^2 )⇒z^2 +(1/z^2 )=w^2 −2  w^2 −2+3w+2=0  ⇒w^2 +3w=0⇒w=0;−3  ⇒z+(1/z)=0⇒z^2 +1=0⇒z_(1,2) =±i  z+(1/z)=−3⇒z^2 +3z+1=0⇒z_(3,4) =((−3±(√5))/2)  ⇒z∈{i,−i,(((√5)−3)/2),−((3+(√5))/2)} ; Ω=2
$$\frac{{k}^{\mathrm{3}} −\mathrm{1}}{{k}^{\mathrm{3}} +\mathrm{1}}=\frac{\left({k}−\mathrm{1}\right)\left({k}^{\mathrm{2}} +{k}+\mathrm{1}\right)}{\left({k}+\mathrm{1}\right)\left({k}^{\mathrm{2}} −{k}+\mathrm{1}\right)} \\ $$$$\underset{{k}=\mathrm{2}} {\overset{{m}} {\prod}}\frac{{k}^{\mathrm{3}} −\mathrm{1}}{{k}^{\mathrm{3}} +\mathrm{1}}=\underset{{k}=\mathrm{2}} {\overset{{m}} {\prod}}\frac{{k}−\mathrm{1}}{{k}+\mathrm{1}}\underset{{k}=\mathrm{2}} {\overset{{m}} {\prod}}\frac{{k}^{\mathrm{2}} +{k}+\mathrm{1}}{{k}^{\mathrm{2}} −{k}+\mathrm{1}} \\ $$$$\underset{{k}=\mathrm{2}} {\overset{{m}} {\prod}}\frac{{k}^{\mathrm{2}} +{k}+\mathrm{1}}{{k}^{\mathrm{2}} −{k}+\mathrm{1}}\:=\:\frac{\underset{{k}=\mathrm{2}} {\overset{{m}} {\prod}}{k}^{\mathrm{2}} +{k}+\mathrm{1}}{\underset{{k}=\mathrm{2}} {\overset{{m}} {\prod}}{k}^{\mathrm{2}} −{k}+\mathrm{1}}\:= \\ $$$$=\frac{\underset{{k}=\mathrm{2}} {\overset{{m}} {\prod}}{k}^{\mathrm{2}} +{k}+\mathrm{1}}{\underset{{k}=\mathrm{2}} {\overset{{m}} {\prod}}\left({k}−\mathrm{1}\right)^{\mathrm{2}} +\left({k}−\mathrm{1}\right)+\mathrm{1}}= \\ $$$$=\frac{\underset{{k}=\mathrm{2}} {\overset{{m}} {\prod}}{k}^{\mathrm{2}} +{k}+\mathrm{1}}{\underset{{k}=\mathrm{1}} {\overset{{m}−\mathrm{1}} {\prod}}{k}^{\mathrm{2}} +{k}+\mathrm{1}}=\frac{{m}^{\mathrm{2}} +{m}+\mathrm{1}}{\mathrm{3}} \\ $$$$\underset{{k}=\mathrm{2}} {\overset{{m}} {\prod}}\frac{{k}−\mathrm{1}}{{k}+\mathrm{1}}=\underset{{k}=\mathrm{2}} {\overset{{m}} {\prod}}\frac{{k}−\mathrm{1}}{{k}}\underset{{k}=\mathrm{2}} {\overset{{m}} {\prod}}\frac{{k}}{{k}+\mathrm{1}}= \\ $$$$=\frac{\mathrm{1}}{{m}}\:\frac{\mathrm{2}}{{m}+\mathrm{1}}=\frac{\mathrm{2}}{{m}\left({m}+\mathrm{1}\right)} \\ $$$$\underset{{k}=\mathrm{2}} {\overset{{m}} {\prod}}\frac{{k}^{\mathrm{3}} −\mathrm{1}}{{k}^{\mathrm{3}} +\mathrm{1}}=\frac{\mathrm{2}\left({m}^{\mathrm{2}} +{m}+\mathrm{1}\right)}{\mathrm{3}{m}\left({m}+\mathrm{1}\right)} \\ $$$$\Rightarrow\underset{{k}=\mathrm{2}} {\overset{\infty} {\prod}}\frac{{k}^{\mathrm{3}} −\mathrm{1}}{{k}^{\mathrm{3}} +\mathrm{1}}=\underset{{m}\rightarrow\infty} {{lim}}\frac{\mathrm{2}\left({m}^{\mathrm{2}} +{m}+\mathrm{1}\right)}{\mathrm{3}{m}\left({m}+\mathrm{1}\right)}=\frac{\mathrm{2}}{\mathrm{3}} \\ $$$$\Rightarrow\Omega=\underset{{n}=\mathrm{1}} {\overset{\infty} {\sum}}\left(\frac{\mathrm{2}}{\mathrm{3}}\right)^{{n}} =\frac{\mathrm{2}}{\mathrm{3}}\underset{{n}=\mathrm{0}} {\overset{\infty} {\sum}}\left(\frac{\mathrm{2}}{\mathrm{3}}\right)^{{n}} = \\ $$$$=\frac{\mathrm{2}}{\mathrm{3}}\:\frac{\mathrm{1}}{\mathrm{1}−\frac{\mathrm{2}}{\mathrm{3}}}=\mathrm{2} \\ $$$${z}^{\mathrm{4}} +\mathrm{3}{z}^{\mathrm{3}} +\Omega{z}^{\mathrm{2}} +\mathrm{3}{z}+\mathrm{1}=\mathrm{0} \\ $$$${z}^{\mathrm{4}} +\mathrm{3}{z}^{\mathrm{3}} +\mathrm{2}{z}^{\mathrm{2}} +\mathrm{3}{z}+\mathrm{1}=\mathrm{0} \\ $$$${z}=\mathrm{0}\Rightarrow{not}\:{sol}. \\ $$$${z}^{\mathrm{2}} +\mathrm{3}{z}+\mathrm{2}+\mathrm{3}\frac{\mathrm{1}}{{z}}+\frac{\mathrm{1}}{{z}^{\mathrm{2}} }=\mathrm{0} \\ $$$${w}={z}+\frac{\mathrm{1}}{{z}}\Rightarrow{w}^{\mathrm{2}} =\mathrm{2}+{z}^{\mathrm{2}} +\frac{\mathrm{1}}{{z}^{\mathrm{2}} }\Rightarrow{z}^{\mathrm{2}} +\frac{\mathrm{1}}{{z}^{\mathrm{2}} }={w}^{\mathrm{2}} −\mathrm{2} \\ $$$${w}^{\mathrm{2}} −\mathrm{2}+\mathrm{3}{w}+\mathrm{2}=\mathrm{0} \\ $$$$\Rightarrow{w}^{\mathrm{2}} +\mathrm{3}{w}=\mathrm{0}\Rightarrow{w}=\mathrm{0};−\mathrm{3} \\ $$$$\Rightarrow{z}+\frac{\mathrm{1}}{{z}}=\mathrm{0}\Rightarrow{z}^{\mathrm{2}} +\mathrm{1}=\mathrm{0}\Rightarrow{z}_{\mathrm{1},\mathrm{2}} =\pm{i} \\ $$$${z}+\frac{\mathrm{1}}{{z}}=−\mathrm{3}\Rightarrow{z}^{\mathrm{2}} +\mathrm{3}{z}+\mathrm{1}=\mathrm{0}\Rightarrow{z}_{\mathrm{3},\mathrm{4}} =\frac{−\mathrm{3}\pm\sqrt{\mathrm{5}}}{\mathrm{2}} \\ $$$$\Rightarrow{z}\in\left\{{i},−{i},\frac{\sqrt{\mathrm{5}}−\mathrm{3}}{\mathrm{2}},−\frac{\mathrm{3}+\sqrt{\mathrm{5}}}{\mathrm{2}}\right\}\:;\:\Omega=\mathrm{2} \\ $$
Commented by Shrinava last updated on 28/Feb/23
perfect dear professor thank you
$$\mathrm{perfect}\:\mathrm{dear}\:\mathrm{professor}\:\mathrm{thank}\:\mathrm{you} \\ $$

Leave a Reply

Your email address will not be published. Required fields are marked *